Last visit was: 23 Apr 2024, 18:13 It is currently 23 Apr 2024, 18:13

Close
GMAT Club Daily Prep
Thank you for using the timer - this advanced tool can estimate your performance and suggest more practice questions. We have subscribed you to Daily Prep Questions via email.

Customized
for You

we will pick new questions that match your level based on your Timer History

Track
Your Progress

every week, we’ll send you an estimated GMAT score based on your performance

Practice
Pays

we will pick new questions that match your level based on your Timer History
Not interested in getting valuable practice questions and articles delivered to your email? No problem, unsubscribe here.
Close
Request Expert Reply
Confirm Cancel
SORT BY:
Date
Tags:
Show Tags
Hide Tags
User avatar
Manager
Manager
Joined: 28 Jul 2011
Posts: 225
Own Kudos [?]: 1372 [31]
Given Kudos: 16
Location: United States
Concentration: International Business, General Management
GPA: 3.86
WE:Accounting (Commercial Banking)
Send PM
Most Helpful Reply
avatar
Intern
Intern
Joined: 31 Jul 2011
Posts: 9
Own Kudos [?]: 15 [11]
Given Kudos: 7
Send PM
General Discussion
User avatar
Manager
Manager
Joined: 13 Aug 2010
Posts: 88
Own Kudos [?]: 120 [0]
Given Kudos: 1
Send PM
User avatar
Manager
Manager
Joined: 28 Jul 2011
Posts: 225
Own Kudos [?]: 1372 [1]
Given Kudos: 16
Location: United States
Concentration: International Business, General Management
GPA: 3.86
WE:Accounting (Commercial Banking)
Send PM
Re: Two computer companies, Garnet and Renco, each pay Salcor to provide h [#permalink]
1
Kudos
prab wrote:
I came close two answers C and D.....and the answer is C .....but Can anyone please explain why the answer is not C.......

?????

the answer is C or the answer is C ?


Sorry it was a Typo error the answer is C but i marked E......
User avatar
Manager
Manager
Joined: 13 Aug 2010
Posts: 88
Own Kudos [?]: 120 [1]
Given Kudos: 1
Send PM
Re: Two computer companies, Garnet and Renco, each pay Salcor to provide h [#permalink]
1
Kudos
i might be mistaken but, isn't the option C only the option in context here.
User avatar
Manager
Manager
Joined: 25 Nov 2011
Posts: 126
Own Kudos [?]: 870 [1]
Given Kudos: 20
Location: India
Concentration: Technology, General Management
GPA: 3.95
WE:Information Technology (Computer Software)
Send PM
Re: Two computer companies, Garnet and Renco, each pay Salcor to provide h [#permalink]
1
Kudos
prab wrote:
i might be mistaken but, isn't the option C only the option in context here.


Even A is also a contender because of the word 'possibility' --> stroke MAY or MAY NOT occur. If it occurs, it strengths the statement otherwise it weakens.
Re: Two computer companies, Garnet and Renco, each pay Salcor to provide h [#permalink]
2
Kudos
1
Bookmarks
no, option C actually is in context and does weaken.
The argument is this: SaleCor's financial incentive to volunteer for Garnet's employees = SaleCor can save costs if Garnet's employees don't end up with serious ailments because of untreated cholesterol (remember Insurer has to pay for the expensive ailments too - which are more expensive to pay for than the cholesterol check/early treatment)
Whereas this financial incentive is non-existent for SaleCor in the case of RenCo because RenCo's employees leave early.
So, the assumption here is: 1) when employees with cholesterol are untreated for long = expensive ailments arise
2) RenCo employees don't stay long = no expensive ailments arise for SaleCor to pay (so why should SaleCor now waste money to sponsor for cholesterol check if these employees are not going to stay for a long enough period to incur SaleCor costs through expensive ailments = No FINANCIAL INCENTIVE for SaleCor)

Weaken:
SaleCor's reasoning seems fair enough. But when would it be counterproductive for SaleCor.
1) assume these employees leaving RenCo (who has SaleCor as its insurer) end up with another company for which SaleCor is an insurer. Now SaleCor will end up paying for these employees (=who did not have the cholesterol check and are therefore more likely to end up with serious ailments which incur huge costs for the insurer). So in this case SaleCor still has the financial incentive to pay for those cholesterol checks
2) a less probable scenario - when some study finds all the RenCo employees are unhealthy and say 90% of them contract those ailments during the short time they are with RenCo and SaleCor has to pay for those treatments. So in this case SaleCor still has the financial incentive to pay for those cholesterol checks

Also, we are only weakening the fact that RenCo's employees short stay at company = NO financial incentive for SaleCor.
So the answer must weaken this aspect only.
(A) means no fin incentive for paying for Garnet too. it falsifies the unstated premise - "cholesterol checks & early treatment does help"
(B) this only strengthens the case for SaleCor to an extent if those people talked about in the answer choice includes RenCo employees
(C) a re-worded case I came up with for Weakening the argument
(D) no impact on argument. but to the possible extent this can relate to the argument - same number of employees means SaleCor is saving as much from RenCo as it spends for Garner. nothing useful can be inferred from this info
(E) note old age cannot mean higher cholesterol - nothing of this sort is mentioned. plus even if age is correlated with high cholesterol and strokes, the argument kind of strengthens SaleCor's case for RenCo.

hope this helps!
avatar
Intern
Intern
Joined: 28 Jun 2015
Posts: 18
Own Kudos [?]: 17 [0]
Given Kudos: 42
Location: United States
GMAT 1: 610 Q49 V27
GPA: 2.5
WE:Project Management (Telecommunications)
Send PM
Re: Two computer companies, Garnet and Renco, each pay Salcor to provide h [#permalink]
C is a sufficient one:
It is possible that employees who jumper from R to G didn't receive any early treatment of cholesterol and worked for a long time.
This situation according to the argument ( strokes would occur several years later) will make the insurer be incentive.

E actually strengthens the argument.
VP
VP
Joined: 12 Dec 2016
Posts: 1030
Own Kudos [?]: 1779 [0]
Given Kudos: 2562
Location: United States
GMAT 1: 700 Q49 V33
GPA: 3.64
Send PM
Re: Two computer companies, Garnet and Renco, each pay Salcor to provide h [#permalink]
A,D,E are out of scope because these options discuss something else that are not covered in the argument, and also, all of these questions do not relate to both companies.
B does not address the issue in the argument, then it is C even though C is quite confusing, and difficult to understand.
Manager
Manager
Joined: 12 Feb 2015
Posts: 97
Own Kudos [?]: 166 [2]
Given Kudos: 9
Location: India
Concentration: Technology, Operations
GMAT 1: 680 Q50 V31
GPA: 3.9
WE:Information Technology (Computer Software)
Send PM
Re: Two computer companies, Garnet and Renco, each pay Salcor to provide h [#permalink]
2
Kudos
chesstitans wrote:
A,D,E are out of scope because these options discuss something else that are not covered in the argument, and also, all of these questions do not relate to both companies.
B does not address the issue in the argument, then it is C even though C is quite confusing, and difficult to understand.


Agree C is a bit confusing, but is the right answer.

(A) Early treatment of high cholesterol does not eliminate the possibility of a stroke later in life.
----we already know this. Not a weakener.

(B) People often obtain early treatment for high cholesterol on their own.
----so what, doesnt weaken the fact about renco

(C) Garnet hires a significant number of former employees of Renco.
----This weakens the conclusion. If renco people are getting hired by garnet, then garnet will have to do the checkup s for them, As renco is not doing. So, if Renco people also go for checkups, that would be beneficial for Salcor. (it can extract double revenue by checkup of same employee both at renco as well as at garnet).

(D) Renco and Garnet have approximately the same number of employees.
----so what, nothing to do with weakening of renco fact

(E) Renco employees are not, on average, significantly younger than Garnet employees.
----that actually strengthens the argument by saying that renco's employees are a bit younger than garnet's and so, even if they dont go for checkups, that shouldn't harm much.

==========Hit +1 KUDOS if you liked my post================
VP
VP
Joined: 12 Dec 2016
Posts: 1030
Own Kudos [?]: 1779 [0]
Given Kudos: 2562
Location: United States
GMAT 1: 700 Q49 V33
GPA: 3.64
Send PM
Re: Two computer companies, Garnet and Renco, each pay Salcor to provide h [#permalink]
I believe C shows a reverse of the causal relationship.
the key word "several years later" means the longer employees stay at Garnet, the higher risk of strokes, and thus Garnet offers medical incentives.
same to Renco; if employees leave, then Renco will not have to offer incentives.

C tells that no incentives from Renco cause employees to leave to a better company.
Intern
Intern
Joined: 07 Jun 2018
Posts: 9
Own Kudos [?]: 1 [0]
Given Kudos: 54
Location: India
Concentration: Other, Operations
GPA: 3.15
WE:General Management (Computer Software)
Send PM
Re: Two computer companies, Garnet and Renco, each pay Salcor to provide h [#permalink]
Can someone please explain why option A is ruled out. It is a clear weakener while option C needs an assumption that the existing employees of Garnet are not leaving the organization.

Also, there is an another assumption(if we choose option C)that employees of Garnet are not moving to Renco.

Not sure why OA is 'C' in this question
Intern
Intern
Joined: 13 Aug 2018
Posts: 37
Own Kudos [?]: 7 [0]
Given Kudos: 118
Send PM
Re: Two computer companies, Garnet and Renco, each pay Salcor to provide h [#permalink]
I do not understand how answer C can weaken the conclusion: Salcor lacks any financial incentive to provide similar encouragement to Renco employees

Please help
Director
Director
Joined: 28 Sep 2018
Posts: 734
Own Kudos [?]: 558 [0]
Given Kudos: 248
GMAT 1: 660 Q48 V33 (Online)
GMAT 2: 700 Q49 V37
Send PM
Re: Two computer companies, Garnet and Renco, each pay Salcor to provide h [#permalink]
EducationAisle while I understand why (C) is the answer. Please could you throw some light on (E) had it been "the employees on average are "older"? would that weaken the argument? or have no effect?
CEO
CEO
Joined: 27 Mar 2010
Posts: 3675
Own Kudos [?]: 3528 [0]
Given Kudos: 149
Location: India
Schools: ISB
GPA: 3.31
Send PM
Re: Two computer companies, Garnet and Renco, each pay Salcor to provide h [#permalink]
Expert Reply
Hoozan wrote:
EducationAisle while I understand why (C) is the answer. Please could you throw some light on (E) had it been "the employees on average are "older"? would that weaken the argument? or have no effect?

Would agree Hoozan; in that case, E would have been a potential answer choice.
Director
Director
Joined: 28 Sep 2018
Posts: 734
Own Kudos [?]: 558 [0]
Given Kudos: 248
GMAT 1: 660 Q48 V33 (Online)
GMAT 2: 700 Q49 V37
Send PM
Re: Two computer companies, Garnet and Renco, each pay Salcor to provide h [#permalink]
EducationAisle wrote:
Hoozan wrote:
EducationAisle while I understand why (C) is the answer. Please could you throw some light on (E) had it been "the employees on average are "older"? would that weaken the argument? or have no effect?

Would agree Hoozan; in that case, E would have been a potential answer choice.


Could you explain how the modified version of (E) would weaken?
CEO
CEO
Joined: 27 Mar 2010
Posts: 3675
Own Kudos [?]: 3528 [0]
Given Kudos: 149
Location: India
Schools: ISB
GPA: 3.31
Send PM
Re: Two computer companies, Garnet and Renco, each pay Salcor to provide h [#permalink]
Expert Reply
Hoozan wrote:
Could you explain how the modified version of (E) would weaken?

Well, if all/most employees of Renco are significantly old (say in their 50s), then it does make sense for Salcor to encourage Renco employees to have their cholesterol levels tested and to obtain early treatment for high cholesterol, despite the fact that Renco employees generally remain with Renco only for a few years.

So, the modified version of E is weakening the argument.
Manager
Manager
Joined: 15 Aug 2020
Posts: 148
Own Kudos [?]: 52 [0]
Given Kudos: 16
Send PM
Re: Two computer companies, Garnet and Renco, each pay Salcor to provide h [#permalink]
CONCLUSION - Salcor lacks incentive to provide encouragement to Renco employees

GIVEN -

1) Early treatment of high cholesterol prevents strokes after several years
2) Garnet employees encouraged to get cholesterol checked
3) Renco employees - remain only for few years

WEAKNER - What new information reduces the belief in the conclusion?

ANSWER CHOICE ANALYSIS -

A) Irrelevant to conclusion
B) Not related to conclusion
C) CORRECT - As the employees shift from Renco to Garnet, incentives are being given
D) Doesn't matter
E) Doesn't matter as no specific age is given
e-GMAT Representative
Joined: 02 Nov 2011
Posts: 4341
Own Kudos [?]: 30776 [1]
Given Kudos: 632
GMAT Date: 08-19-2020
Send PM
Re: Two computer companies, Garnet and Renco, each pay Salcor to provide h [#permalink]
1
Kudos
Expert Reply
mydreammba wrote:
Two computer companies, Garnet and Renco, each pay Salcor to provide health insurance for their employees. Because early treatment of high cholesterol can prevent strokes that would otherwise occur several years later, Salcor encourages Garnet employees to have their cholesterol levels tested and to obtain early treatment for high cholesterol. Renco employees generally remain with Renco only for a few years, however. Therefore, Salcor lacks any financial incentive to provide similar encouragement to Renco employees.

Which of the following, if true, most seriously weakens the argument?

(A) Early treatment of high cholesterol does not eliminate the possibility of a stroke later in life.

(B) People often obtain early treatment for high cholesterol on their own.

(C) Garnet hires a significant number of former employees of Renco.

(D) Renco and Garnet have approximately the same number of employees.

(E) Renco employees are not, on average, significantly younger than Garnet employees.

Attachment:
CR1.jpg


SOLUTION

Passage Analysis

Two computer companies, Garnet and Renco, each pay Salcor to provide health insurance for their employees.
• Salcor(S) provides health insurance to the employees of two computer companies- Garnet (G) and Renco (R)
Because early treatment of high cholesterol can prevent strokes that would otherwise occur several years later, Salcor encourages Garnet employees to have their cholesterol levels tested and to obtain early treatment for high cholesterol
• S encourages G employees to test cholesterol levels and get early treatment for high cholesterol.
• This is because early treatment of high cholesterol can prevent strokes due to these from happening. (If strokes happen the insurance providers will have to pay for the expensive treatment)

Renco employees generally remain with Renco only for a few years, however.
Employees of R usually stay with R only for a few years. So, the likelihood of them having a stroke during their employment in R is very low.
Therefore, Salcor lacks any financial incentive to provide similar encouragement to Renco employees.
Thus, S does not have any financial gains in encouraging R employees to check their cholesterol levels and take early treatment.

Conclusion: Salcor lacks any financial incentive to provide similar encouragement to Renco employees.

Question stem Analysis

Which of the following, if true, most seriously weakens the argument?
We need to find the statement that weakens the given conclusion.

Prethinking

Weakener framework

What new information will make us believe less that Salcor lacks any financial incentive to provide similar encouragement to Renco employees?
Given that
• early treatment of high cholesterol can prevent strokes that would otherwise occur several years later
• Salcor encourages Garnet employees to have their cholesterol levels tested and to obtain early treatment for high cholesterol.
• Renco employees generally remain with Renco only for a few years.

Weakener 1- Due to changes in lifestyle, the gap in years between diagnosis of high cholesterol and occurrence of stroke is decreasing. If this is true, then R employees with high cholesterol may get strokes even when they are working with R. This weakens our belief in the conclusion that S lacks any financial incentive to provide encouragement to R employees.

Weakener 2- A large number of employees who leave R go to G or another company to which employee insurance is provided by S. If this is true, then it would make sense for S to provide the same encouragement to R employees too. Hence, this weakens our belief in the conclusion.

Option Analysis

(A) Early treatment of high cholesterol does not eliminate the possibility of a stroke later in life.
It is not necessary to eliminate the possibility of stroke, it is good enough to reduce the probability. Also this option does not talk about the employees of R. On the whole, it is an incorrect answer.

(B) People often obtain early treatment for high cholesterol on their own.
This is an irrelevant answer choice since whether people often do it or not, we do not know if the ones with risk of stroke do it. We know that irrespective of this, S is encouraging G employees to get cholesterol levels checked, and that S is not doing the same for R employees. This option does not in any way weaken the conclusion about R employees. Hence it is an incorrect option.

(C) Garnet hires a significant number of former employees of Renco.
This weakener is in line with our prethought weakener 2. Hence it is the right answer.

(D) Renco and Garnet have approximately the same number of employees.
The number of employees is not relevant in this context. Even if R had a very low number of employees, it does not mean S needn’t bother about the expense of Stroke treatment.

(E) Renco employees are not, on average, significantly younger than Garnet employees.
Whether or not Renco employees are younger, if they are diagnosed for high cholesterol, there is a chance for them to have a stroke years later. The impact of age on the likelihood of stroke, in high cholesterol cases is out of scope. Hence this option is irrelevant.

Thus the correct answer is C.
User avatar
Non-Human User
Joined: 01 Oct 2013
Posts: 17206
Own Kudos [?]: 848 [0]
Given Kudos: 0
Send PM
Re: Two computer companies, Garnet and Renco, each pay Salcor to provide h [#permalink]
Hello from the GMAT Club VerbalBot!

Thanks to another GMAT Club member, I have just discovered this valuable topic, yet it had no discussion for over a year. I am now bumping it up - doing my job. I think you may find it valuable (esp those replies with Kudos).

Want to see all other topics I dig out? Follow me (click follow button on profile). You will receive a summary of all topics I bump in your profile area as well as via email.
GMAT Club Bot
Re: Two computer companies, Garnet and Renco, each pay Salcor to provide h [#permalink]
Moderators:
GMAT Club Verbal Expert
6917 posts
GMAT Club Verbal Expert
238 posts
CR Forum Moderator
832 posts

Powered by phpBB © phpBB Group | Emoji artwork provided by EmojiOne